Difference between revisions of "2010 AMC 12A Problems/Problem 14"

(Solution 2(Trick))
(Solution)
 
(5 intermediate revisions by 4 users not shown)
Line 7: Line 7:
  
 
== Solution ==
 
== Solution ==
By the [[Angle Bisector Theorem]], we know that <math>\frac{AB}{BC} = \frac{3}{8}</math>. If we use the lowest possible integer values for AB and BC (the measures of AD and DC, respectively), then <math>AB + BC = AD + DC = AC</math>, contradicting the [[Triangle Inequality]]. If we use the next lowest values (<math>AB = 6</math> and <math>BC = 16</math>), the Triangle Inequality is satisfied. Therefore, our answer is <math>6 + 16 + 3 + 8 = \boxed{33}</math>, or choice <math>\textbf{(B)}</math>.
+
By the [[Angle Bisector Theorem]], we know that <math>\frac{AB}{BC} = \frac{3}{8}</math>. If we use the lowest possible integer values for <math>AB</math> and <math>BC</math> (the lengths of <math>AD</math> and <math>DC</math>, respectively), then <math>AB + BC = AD + DC = AC</math>, contradicting the [[Triangle Inequality]]. If we use the next lowest values (<math>AB = 6</math> and <math>BC = 16</math>), the Triangle Inequality is satisfied. Therefore, our answer is <math>6 + 16 + 3 + 8 = \boxed{33}</math>, or choice <math>\textbf{(B)}</math>.
  
 
== Solution 2(Trick) ==
 
== Solution 2(Trick) ==
We find that <math>\frac{AB}{BC}=\frac{3}{8}</math> by the [[Angle Bisector Theorem]] so we let the lengths be <math>3n</math> and <math>8n</math>, respectively where <math>n</math> is a positive integer. Also since <math>AD=3</math> and <math>BC=8</math>, we notice that the perimeter of the triangle is the sum of these, namely <math>3n+8n+3+8=11n+11.</math> This can be factored into <math>11(n+1)</math> and so the sum must be a multiple of <math>11</math>. The only answer choice which is a multiple of <math>11</math> is <math>\boxed{\textbf{(B)}11}</math>. ~mathboy282
+
We find that <math>\frac{AB}{BC}=\frac{3}{8}</math> by the [[Angle Bisector Theorem]] so we let the lengths be <math>3n</math> and <math>8n</math>, respectively where <math>n</math> is a positive integer. Also since <math>AD=3</math> and <math>BC=8</math>, we notice that the perimeter of the triangle is the sum of these, namely <math>3n+8n+3+8=11n+11.</math> This can be factored into <math>11(n+1)</math> and so the sum must be a multiple of <math>11</math>. The only answer choice which is a multiple of <math>11</math> is <math>\boxed{\textbf{(B)} 33}</math>. ~mathboy282
  
==Video Solution==
+
==Video Solution by the Beauty of Math==
 
https://youtu.be/rsURe5Xh-j0
 
https://youtu.be/rsURe5Xh-j0
 
~IceMatrix
 
  
 
== See also ==
 
== See also ==
 +
{{AMC10 box|year=2010|ab=A|num-b=15|num-a=17}}
 
{{AMC12 box|year=2010|num-b=13|num-a=15|ab=A}}
 
{{AMC12 box|year=2010|num-b=13|num-a=15|ab=A}}
  
 
[[Category:Introductory Geometry Problems]]
 
[[Category:Introductory Geometry Problems]]
 
{{MAA Notice}}
 
{{MAA Notice}}

Latest revision as of 20:29, 4 July 2021

The following problem is from both the 2010 AMC 12A #14 and 2010 AMC 10A #16, so both problems redirect to this page.

Problem

Nondegenerate $\triangle ABC$ has integer side lengths, $\overline{BD}$ is an angle bisector, $AD = 3$, and $DC=8$. What is the smallest possible value of the perimeter?

$\textbf{(A)}\ 30 \qquad \textbf{(B)}\ 33 \qquad \textbf{(C)}\ 35 \qquad \textbf{(D)}\ 36 \qquad \textbf{(E)}\ 37$

Solution

By the Angle Bisector Theorem, we know that $\frac{AB}{BC} = \frac{3}{8}$. If we use the lowest possible integer values for $AB$ and $BC$ (the lengths of $AD$ and $DC$, respectively), then $AB + BC = AD + DC = AC$, contradicting the Triangle Inequality. If we use the next lowest values ($AB = 6$ and $BC = 16$), the Triangle Inequality is satisfied. Therefore, our answer is $6 + 16 + 3 + 8 = \boxed{33}$, or choice $\textbf{(B)}$.

Solution 2(Trick)

We find that $\frac{AB}{BC}=\frac{3}{8}$ by the Angle Bisector Theorem so we let the lengths be $3n$ and $8n$, respectively where $n$ is a positive integer. Also since $AD=3$ and $BC=8$, we notice that the perimeter of the triangle is the sum of these, namely $3n+8n+3+8=11n+11.$ This can be factored into $11(n+1)$ and so the sum must be a multiple of $11$. The only answer choice which is a multiple of $11$ is $\boxed{\textbf{(B)} 33}$. ~mathboy282

Video Solution by the Beauty of Math

https://youtu.be/rsURe5Xh-j0

See also

2010 AMC 10A (ProblemsAnswer KeyResources)
Preceded by
Problem 15
Followed by
Problem 17
1 2 3 4 5 6 7 8 9 10 11 12 13 14 15 16 17 18 19 20 21 22 23 24 25
All AMC 10 Problems and Solutions
2010 AMC 12A (ProblemsAnswer KeyResources)
Preceded by
Problem 13
Followed by
Problem 15
1 2 3 4 5 6 7 8 9 10 11 12 13 14 15 16 17 18 19 20 21 22 23 24 25
All AMC 12 Problems and Solutions

The problems on this page are copyrighted by the Mathematical Association of America's American Mathematics Competitions. AMC logo.png